2002 AMC 10B Problems/Problem 7

Revision as of 02:46, 27 December 2008 by Lifeisacircle (talk | contribs) (still needs solution)
(diff) ← Older revision | Latest revision (diff) | Newer revision → (diff)

Problem

Let $n$ be a positive integer such that $\frac {1}{2} + \frac {1}{3} + \frac {1}{7} + \frac {1}{n}$ is an integer. Which of the following statements is [b]not[/b] true?

$\mathrm{(A) \ } 2\text{ divides }n\qquad \mathrm{(B) \ } 3\text{ divides }n\qquad \mathrm{(C) \ } 6\text{ divides }n\qquad \mathrm{(D) \ } 7\text{ divides }n\qquad \mathrm{(E) \ } n>84$